4

The following passage has been extracted from the book "Mathematical methods for Physicists":

A key idea of the present chapter is that a quantity that is properly called a vector must have the transformation properties that preserve its essential features under coordinate transformation; there exist quantities with direction and magnitude that do not transform appropriately and hence are not vectors.

Cross product: $\nabla \times (Vector)=Vector$

From the above equation of cross product we can say that $\nabla$ is a vector (specifically vector operator). However, a vector generally has magnitude and an associated direction. While in case of $\nabla$, it might satisfy essential features under transformation to be a vector, but I don't see whether it has magnitude or not? Does it has magnitude? If so, what is it? Or otherwise is it that a vector need not have magnitude?

Sensebe
  • 915
  • Google "nabla is not a vector". – Tony Piccolo Nov 19 '14 at 16:51
  • I did it before posting. When I did I found no reliable results. Did you find any? – Sensebe Nov 19 '14 at 16:55
  • 1
    As in every branch of mathematics, one should define $X$ before asking "is $Y$ an example of $X$?". If you find yourself unwilling to do this, then you are instead asking the question "Should we consider $Y$ an example of $X$, and thus formulate our definition of $X$ accordingly?". I think you meant to ask the latter but accidentally phrased it as the former. – RghtHndSd Nov 19 '14 at 17:48
  • Who are the authors of the book? – Jonas Meyer Nov 19 '14 at 18:17
  • 2
    @JonasMeyer: Arfken, Weber, and Harris. – Sensebe Nov 20 '14 at 00:53
  • 1
    Read The Feynman Lectures on Physics second volume, second chapter, sections 4 and 5. – Tony Piccolo Nov 20 '14 at 08:01
  • @TonyPiccolo: To be frank, I read it before posting. He has made the analysis, but he didn't reach where I expected him to reach. – Sensebe Nov 20 '14 at 14:42
  • 1
    In general, vectors need not have "magnitude and direction". The mathematical concept of a vector is just anything that you can add with similar things and multiply by a constant. – Ben Grossmann Nov 20 '14 at 15:47
  • 1
    @Omnomnomnom: vectors need not have "magnitude and direction" What is the source for your statement? – Sensebe Nov 20 '14 at 16:06
  • 1
    I'm not sure any one source will do here, but the gist is this: "direction" requires a notion of angle between two vectors, which requires an inner product. Most normed vector spaces (vector spaces with "magnitude") are not inner product spaces (vector spaces with "direction"), and only some normed vector spaces can be assigned an inner product that makes sense. – Ben Grossmann Nov 20 '14 at 16:17
  • 1
    In fact, not all vector spaces have a notion of "magnitude" (i.e. a norm) that makes sense. So, we can indeed have vectors where no concept of either magnitude or direction "fits" appropriately. – Ben Grossmann Nov 20 '14 at 16:21
  • 1
    The bright side of this development is that it may yet be possible to conceptualize $\nabla$ as some sort of "vector". However, it is clear that in such a structure, the usual notion of "length" falls apart, since (as you seem to have inferred) no value of $|\nabla|$ really makes sense here. – Ben Grossmann Nov 20 '14 at 16:24
  • Did you see this question? : http://math.stackexchange.com/questions/1032382/what-is-the-product-of-magnitudes-frac-partial-partial-x-and-x – Sensebe Nov 21 '14 at 16:35

5 Answers5

11

Let $f=f(x,y,z)$ be a scalar function and $\mathbf F=\langle F_1(x,y,z),F_2(x,y,z),F_3(x,y,z)\rangle$ be a vector field in $\mathbb{R}^3$. Then we can think of $f$ or $\mathbf F$ (as appropriate) as the inputs to the operators grad, div, curl, and even laplacian with the resulting outputs indicated:

\begin{align} f\longrightarrow &\ \color{blue}{{\LARGE\boxed{\nabla}}} \longrightarrow \langle f_x,f_y,f_z\rangle\\ \mathbf F\longrightarrow &\ \color{blue}{{\LARGE\boxed{\nabla\cdot}}} \longrightarrow {\partial F_1\over \partial x}+{\partial F_2\over \partial y}+{\partial F_3\over \partial z}\\ \mathbf F\longrightarrow &\ \color{blue}{{\LARGE\boxed{\nabla\times}}} \longrightarrow \left\langle {\partial F_3\over \partial y}-{\partial F_2\over \partial z},{\partial F_1\over \partial z}-{\partial F_3\over \partial x},{\partial F_2\over \partial x}-{\partial F_1\over \partial y}\right\rangle\\ f\longrightarrow &\ \color{blue}{{\LARGE\boxed{\nabla\cdot\nabla}}} \longrightarrow f_{xx}+f_{yy}+f_{zz}.\\ \end{align}

Thus $\nabla$ is not a vector, but rather indicates an operator whose action on the input $f$ results in the output $\langle f_x,f_y,f_z\rangle$. Similarly for the others.

If you find the del notation counterproductive, just abandon that notation/nomenclature for this:

\begin{align} f\longrightarrow &\ \color{blue}{{\LARGE\boxed{\text{grad}}}} \longrightarrow \langle f_x,f_y,f_z\rangle\\ \mathbf F\longrightarrow &\ \color{blue}{{\LARGE\boxed{\text{div}}}} \longrightarrow {\partial F_1\over \partial x}+{\partial F_2\over \partial y}+{\partial F_3\over \partial z}\\ \mathbf F\longrightarrow &\ \color{blue}{{\LARGE\boxed{\text{curl}}}} \longrightarrow \left\langle {\partial F_3\over \partial y}-{\partial F_2\over \partial z},{\partial F_1\over \partial z}-{\partial F_3\over \partial x},{\partial F_2\over \partial x}-{\partial F_1\over \partial y}\right\rangle\\ f\longrightarrow &\ \color{blue}{{\LARGE\boxed{\text{lap}}}} \longrightarrow f_{xx}+f_{yy}+f_{zz}.\\ \end{align}

JohnD
  • 14,392
  • 1
  • "∇ is not a vector"-Thank you for the answer. To be frank, I didn't understand your point. I still have trouble in understanding. You see if $\nabla$ is not a vector, why do we use in vector product?
  • – Sensebe Nov 20 '14 at 01:06
  • 1
  • Daust argued $\nabla \times$ to be curl operator and said $\nabla \times f$ not to be cross product. He is interpreting it in that way, but I am interpreting the same thing to be cross product. Whatever may be the interpretation, we must get respective expected significance. With whom do you agree? Here if (you agree with me) $\nabla$ is a vector, then what is its magnitude? Please answer the questions pointing 1) and 2) to maintain order.
  • – Sensebe Nov 20 '14 at 01:12
  • 1
    @Godparticle JohnD's answer is consistent with Daust's argument and inconsistent with yours. – Ben Grossmann Nov 20 '14 at 15:50